2014 dxdy logo

Научный форум dxdy

Математика, Физика, Computer Science, Machine Learning, LaTeX, Механика и Техника, Химия,
Биология и Медицина, Экономика и Финансовая Математика, Гуманитарные науки


Правила форума


В этом разделе нельзя создавать новые темы.

Если Вы хотите задать новый вопрос, то не дописывайте его в существующую тему, а создайте новую в корневом разделе "Помогите решить/разобраться (М)".

Если Вы зададите новый вопрос в существующей теме, то в случае нарушения оформления или других правил форума Ваше сообщение и все ответы на него могут быть удалены без предупреждения.

Не ищите на этом форуме халяву, правила запрещают участникам публиковать готовые решения стандартных учебных задач. Автор вопроса обязан привести свои попытки решения и указать конкретные затруднения.

Обязательно просмотрите тему Правила данного раздела, иначе Ваша тема может быть удалена или перемещена в Карантин, а Вы так и не узнаете, почему.



Начать новую тему Ответить на тему
 
 про дельта-функцию (представление в виде интеграла)
Сообщение03.01.2009, 11:27 


19/07/05
243
Здравствуйте, возник следующий вопрос:
Часто дельта-функцию представляют так -
$$\delta(\omega)=\frac{1}{2\pi}\int\limits_{-\infty}^{+\infty}e^{-i\omega t}\,dt$$. Это можно получить из определения дельта-функции, используя прямое и обратное преобразование Фурье.
Но вот как доказать в лоб, что: $$\delta(\omega)=\lim\limits_{T\to+\infty}\frac{1}{2\pi}\int\limits_{-T}^{+T}e^{-i\omega t}\,dt=\lim\limits_{T\to+\infty}\frac{1}{\pi}\frac{\sin T\omega}{\omega}$$. Буду рад подсказкам как это сделать. Спасибо :wink:

 Профиль  
                  
 
 Re: про дельта-функцию
Сообщение03.01.2009, 11:36 
Аватара пользователя


18/10/08
454
Омск
Zo писал(а):
Здравствуйте, возник следующий вопрос:
Часто дельта-функцию представляют так -
$$\delta(x)=\frac{1}{2\pi}\int\limits_{-\infty}^{+\infty}e^{-i\omega t}\,dt$$.

Непорядок с переменными: слева $x$, справа $\omega$. Должно быть где-то так:
$$\delta(t)=\frac{1}{2\pi}\int\limits_{-\infty}^{+\infty}e^{-i\omega t}\,d\omega$$.

 Профиль  
                  
 
 
Сообщение03.01.2009, 11:45 


19/07/05
243
Спасибо. опечатался я :roll:
так как доказать-то?

 Профиль  
                  
 
 
Сообщение03.01.2009, 12:14 
Экс-модератор


17/06/06
5004
Ну то есть, скажем так, согласно определению обобщенных функций, надо доказать, что последовательность (ну не совсем последовательность, но типа того) функций $f_T:x\mapsto \frac{\sin Tx}x$ является $\delta$-образной, то бишь аппроксимативной единицей. Для этого надо порыться в конспектах по матану и вспомнить какое-нибудь достаточное условие дельтаобразности.

 Профиль  
                  
 
 
Сообщение03.01.2009, 12:20 


19/07/05
243
AD в сообщении #173451 писал(а):
последовательность (ну не совсем последовательность, но типа того) функций $f_T\mapsto \frac{\sin Tx}x$ является $\delta$-образной, то бишь аппроксимативной единицей. Для этого надо порыться в конспектах по матану и вспомнить какое-нибудь достаточное условие дельтаобразности.

а что значит "дельтаобразной...аппроксимативной единице" по рабоче-крестьянски это как будет (не помню, чтобы по матану у нас такое было). Вы не могли бы привести какое-нибудь конкретное достаточное условие?

 Профиль  
                  
 
 
Сообщение03.01.2009, 12:42 
Экс-модератор


17/06/06
5004
Ну, скажем, такую теорему откопал.

Пусть последовательность функций $K_n\in L_1(\mathbb{R})$ такова, что

    1. Нормы $\|K_n\|_1$ ограничены в совокупности
    2. $$\int_{-\infty}^\infty K_n(t)\,dt\xrightarrow[n\to\infty]{}1$$
    3. $\forall\delta>0$ $$\left(\int_{-\infty}^{-\delta}+\int_\delta^\infty\right)|K_n(t)|\,dt\xrightarrow[n\to\infty]{}0$$.

Тогда для любой функции $f\in L_1\cap L_\infty(\mathbb{R})$ имеем

    1. $f*K_n(x)\xrightarrow[n\to\infty]{}f(x)$ для всех точек $x$ непрерывности функции $f$;
    2. Если же $f$ равномерно непрерывна на $\mathbb{R}$, то $f*K_n$ сходятся к $f$ равномерно на $\mathbb{R}$.


Здесь звездочка обозначает свёртку. Но ясно, что применение обобщенной функции всегда можно рассматривать как вычисление значения свертки в одной точке.

Единственная проблема - что у нас $f_T\notin L_1(\mathbb{R})$. Но если обобщенные функции у нас над $\mathcal{D}$, то эту проблему обойти не сложно (То есть рассматривать функции как периодические на отрезке-носителе $f$, и сформулировать такую же теорему для периодических сверток)

 Профиль  
                  
 
 
Сообщение03.01.2009, 12:46 


19/07/05
243
AD, а что означает $\mathcal{D}$ - это какое пространство?

 Профиль  
                  
 
 
Сообщение03.01.2009, 12:50 
Экс-модератор


17/06/06
5004
Думал, что обозначения стандартные. Это пространство финитных бесконечно гладких функций.

Добавлено спустя 43 секунды:

Или, хотите сказать, Вы совсем не в теме, что такое обобщенные функции? :(

 Профиль  
                  
 
 
Сообщение03.01.2009, 13:45 


19/07/05
243
AD в сообщении #173461 писал(а):
Или, хотите сказать, Вы совсем не в теме, что такое обобщенные функции? Sad

ну я про них в "теме" на уровне определения :)
AD, честно говоря я думал, что как-то попроще выводится ответ на мой вопрос, а предложенный Вами план действий меня несколько смущает, потому как вообще не представляю себе как сделать это: "рассматривать функции как периодические на отрезке-носителе $f$, и сформулировать такую же теорему для периодических сверток" :roll: Если бы Вы могли привести мне формулировку хотя бы пункта
AD в сообщении #173457 писал(а):
1. Нормы $\|K_n\|_1$ ограничены в совокупности
в пригодном для моего случая виде, я бы тогда все остальные по аналогии переформулировал бы, а так честно говоря не знаю, с чего и начать. (на всякий случай - я не на мехмате учился :roll: )

 Профиль  
                  
 
 
Сообщение03.01.2009, 14:06 
Экс-модератор


17/06/06
5004
Не, надо, думаю, не пользоваться готовой теоремой, а просто провести аналогичные рассуждения. Перекомпилировать, так сказать, теорему под свой случай, выбросив ненужные соображения про свертки и абсолютную интегрируемость на всём $\mathbb{R}$

То есть что значит доказать равенство $$\frac{1}{2\pi}\lim\limits_{T\to+\infty}\frac{\sin T\omega}{\omega}=\delta(\omega)$$?

Это означает доказать *-слабую сходимость этой, извините за выражение, направленности в пространстве обобщенных функций $\mathcal{D}^*(\mathbb{R})$. А по-человечески это означает, что нужно для каждой функции $f\in\mathcal{D}$ (то есть бесконечно дифференцируемой на $\mathbb{R}$ и равной нулю вне некоторого отрезка (то есть "имеющей компактный носитель", "финитной") доказать равенство

$$\frac{1}{2\pi}\lim\limits_{T\to+\infty}\int_{-\infty}^\infty\frac{\sin T\omega}{\omega}f(\omega)\,d\omega=f(0)$$

На самом деле интеграл берется не по всей прямой, а только по отрезку, вне которого $f$ равна нулю (но для каждой $f$ он будет свой).Так что о сходимости интеграла думать не надо (к тому же, все функции непрерывны).

Это я разжевал определение. Ну а чтобы доказать это равенство, надо проделать примерно то же самое, что делается в доказательстве той теоремы. А именно, разбить интеграл на две части: интеграл по $(-\delta(T),\delta(T))$, который при удачном выборе $\delta(T)$ будет стремиться к $f(0)$, и интеграл по двум оставшимся полуосям, который при том же выборе $\delta(T)$ должен, по идее, стремиться к нулю при $T\to\infty$.

Ручное доказательство примерно так и выглядит. И оно будет в две строчки, если нормально ориентироваться в определениях.

Добавлено спустя 2 минуты 40 секунд:

А еще вам поможет вот такая замена:

$$\int \frac{\sin Tx}xf(x)\,dx=\int \frac{\sin y}yf\bigl(\tfrac yT}\bigr)\,dy$$
Видно, что при $T\to\infty$ получается что-то типа $f(0)$, помноженного на интеграл от $\frac{\sin y}y$. Осталось этим грамотно воспользоваться.

 Профиль  
                  
 
 
Сообщение03.01.2009, 14:52 


19/07/05
243
AD в сообщении #173481 писал(а):
$$\int \frac{\sin Tx}xf(x)\,dx=\int \frac{\sin y}yf\bigl(\tfrac yT}\bigr)\,dy$$
Видно, что при $T\to\infty$ получается что-то типа $f(0)$, помноженного на интеграл от $\frac{\sin y}y$. Осталось этим грамотно воспользоваться.

Вот странно, полистав Зорича, все-таки нашел, что:
$$\int\limits_{0}^{+\infty}\frac{\sin y}{y}\,dy=\frac{\pi}{2}$$ (а ведь когда-то точно проходил такую штуку, а сейчас оказалось, что совсем уже забыл про нее).

Но тогда получается, что
$$\lim_{T\to\infty}\int\limits_{-\infty}^{+\infty} \frac{\sin y}yf\bigl(\tfrac yT}\bigr)\,dy=2f(0)\int\limits_{0}^{+\infty}\frac{\sin y}{y}\,dy=\pi f(0)$$ А д.б. ведь в два раза больше ? Нет наверное, я где-то поспешил. счас подумаю как переделать.
О Ура пофиксил ошибку. Все получилось. Спасибо большое, AD (хотя мог бы и сам догадаться - но совсем из головы вылетело, что интеграл от $\frac{\sin x}{x}$ - это интеграл Дирихле :lol: - вот что значит сдал-забыл да еще и 5 лет почти прошло уже :lol:)

 Профиль  
                  
 
 
Сообщение07.01.2009, 18:15 


29/12/08
7
Я немного позанудствую.
То что $$\lim_{T\to\infty}\int\limits_{-\infty}^{+\infty} \frac{\sin
x}xf\bigl(\tfrac xT}\bigr)\,dx= \int\limits_{-\infty}^{+\infty} \frac{\sin
x}x\left[\lim_{T\to\infty}f\bigl(\tfrac xT}\bigr)\right]\,dx$$ ещё следует обосновать.
Способ первый: попробовать доказать это равенство на языке $\varepsilon-\delta$
Способ второй: воспользоваться критерием допустимости перемещения предельного перехода под интеграл
(Фихтенгольц, n.518).

В конечном итоге доказательства как первым так и вторым способом упираются в одну и ту же проблему.
Я пройду весь путь первым способом.

Итак, допустим, что $$f(x)$$ непрерывна и ограничена на $$(-\infty,\infty)$$ и $$\int_{-\infty}^{+\infty}
\Psi(x)f\bigl(\tfrac xT\bigr)dx$$, где $$\Psi(x)=\frac{\sin x}x$$ существует $$\forall T>0$$.

следует показать, что $$\forall\varepsilon>0\;\;\exists T_0:\;\;
T>T_0\Rightarrow\left|\int_{-\infty}^{+\infty}\Psi(x)\left[f\bigl(\tfrac
xT\bigr)-f(0)\right]dx\right|<\varepsilon$$.
Последний интеграл существует, так как существуют интегралы по отдельности. Далее:
$$\left|\int_{-\infty}^{+\infty}\right|=
\left|\int_{-\infty}^{0}+\int_{0}^{\infty}\right|\leq
\left|\int_{-\infty}^{0}\right|+\left|\int_{0}^{\infty}\right|<
\varepsilon$$.
Следовательно, достаточно показать что
$$\forall\varepsilon>0\;\;\exists T_0:\;\;
T>T_0\Rightarrow\left|\int_{0}^{\infty}\Psi(x)\left[f\bigl(\tfrac
xT\bigr)-f(0)\right]dx\right|<\varepsilon$$.
Пишем:
$$\left|\int_{0}^{\infty}\Psi(x)\left[f\bigl(\tfrac xT\bigr)-f(0)\right]dx\right|=
\left|
\int_{0}^{a}\Psi(x)\left[f\bigl(\tfrac xT\bigr)-f(0)\right]dx+
\int_{a}^{\infty}\Psi(x)\left[f\bigl(\tfrac xT\bigr)-f(0)\right]dx
\right|\leq
$$
$$
\leq\left|
\int_{0}^{a}\Psi(x)\left[f\bigl(\tfrac xT\bigr)-f(0)\right]dx
\right|+
\left|
\int_{a}^{\infty}\Psi(x)\left[f\bigl(\tfrac xT\bigr)-f(0)\right]dx
\right|<\varepsilon$$
Надо доказать, что $$\exists A: \;\; a\geq A\Rightarrow
\left|\int_{a}^{\infty}\Psi(x)\left[f\bigl(\tfrac xT\bigr)-f(0)\right]dx\right|<\varepsilon/2$$ одновременно для всех $$T$$.
Пробуем:
$$\left|\int_{a}^{\infty}\Psi(x)\left[f\bigl(\tfrac xT\bigr)-f(0)\right]dx\right|\leq
\int_{a}^{\infty}\left|\Psi(x)\right|\left|\left[f\bigl(\tfrac xT\bigr)-f(0)\right]\right|dx
\leq K\int_{a}^{\infty}\left|\Psi(x)\right|dx<\varepsilon/2$$,
где $$K=\max(f(x)-f(0))$$. Если $$\int_{0}^{\infty}\left|\Psi(x)\right|dx$$ существует, то понятно, что и требуемое $$A$$ существует.
Возьмём $$a=A$$.
Осталось доказать что:
$$\exists T_0:\;\;T>T_0\Rightarrow\left|\int_{0}^{A}\Psi(x)\left[f\bigl(\tfrac xT\bigr)-f(0)\right]dx\right|<\varepsilon/2$$
В силу непрерывности $$f(x)$$ это доказывается элементарно.

И можно было бы считать первоначальную задачу (перенос границы под интеграл) решённой, если бы не один досадный факт:
интеграл $$\int_{0}^{\infty}\left|\Psi(x)\right|dx$$ расходится!!!
Я по-разному пытался обойти этот момент, но безуспешно. Я не сомневаюсь в правильности переноса границы под интеграл, но строго обосновать не получается.
Может кто-то знает, как обойти этот момент???

Спасибо.

 Профиль  
                  
 
 
Сообщение07.01.2009, 19:44 
Экс-модератор


17/06/06
5004
Alien в сообщении #174838 писал(а):
Надо доказать, что $$\exists A: \;\; a\geq A\Rightarrow \left|\int_{a}^{\infty}\Psi(x)\left[f\bigl(\tfrac xT\bigr)-f(0)\right]dx\right|<\varepsilon/2$$ одновременно для всех $$T$$.
То есть доказать равномерную сходимость несобственного интеграла. А это и означает
Alien в сообщении #174838 писал(а):
воспользоваться критерием допустимости перемещения предельного перехода под интеграл
:wink:

Да-да, сейчас что-нибудь придумаю.

 Профиль  
                  
 
 
Сообщение08.01.2009, 13:09 
Экс-модератор


17/06/06
5004
Короче,

я в своих конспектах по матану писал(а):
Признак Абеля. Если
    1. $\int\limits_{[a,b)}\varphi(x,y)\,dx$ сходится равномерно по $y\in[c,d]$,
    2. $\psi(x,y)$ ограничена на $[a,b)\times[c,d]$ и
    3. $\mathop{\mathrm{Var\,}}\limits_{[a,b)}\psi(x,y)$ ограничена на $[c,d]$,
то $\int\limits_{[a,b)}\varphi(x,y)\psi(x,y)\,dx$ тоже равномерно сходится по $y\in[c,d]$
.

У нас $[a,b)=[0,+\infty)$ (или симметрично), $\varphi(x,y)=\frac{\sin x}x$ не зависит от $y$, потому условие 1 выполнено; $\psi(x,y)=f(xy)$ (то есть $y=\frac1T\in[-1,1]=[c,d]$). И $\psi(x,y)$, очевидно, равномерно ограничена, $\mathrm{VB}$шна (т.к. даже $C^{\infty}$), и вариация при горизонтальном растяжении не меняется, то есть условие 3 тоже выполнено.

 Профиль  
                  
Показать сообщения за:  Поле сортировки  
Начать новую тему Ответить на тему  [ Сообщений: 14 ] 

Модераторы: Модераторы Математики, Супермодераторы



Кто сейчас на конференции

Сейчас этот форум просматривают: нет зарегистрированных пользователей


Вы не можете начинать темы
Вы не можете отвечать на сообщения
Вы не можете редактировать свои сообщения
Вы не можете удалять свои сообщения
Вы не можете добавлять вложения

Найти:
Powered by phpBB © 2000, 2002, 2005, 2007 phpBB Group